aileenann
Thanks Received: 227
Atticus Finch
Atticus Finch
 
Posts: 300
Joined: March 10th, 2009
 
This post thanked 1 time.
 
 

PT 47 S 3 Q 16 In the last election, 89 percent

by aileenann Mon Apr 12, 2010 8:18 pm

This is a flaw, so we'll pay special attention to the conclusion - but also keep our eyes peeled for key premises that might be vulnerable to attack. We'll also keep in mind that we're probably looking to identify an assumption made by the author of this argument.

The conclusion in this question is that the content of the news reveals that reporters were affected by their personal biases - and the important premise is that the candidate the reporters voted for (mostly) received much less proportional criticism than did the other candidate. Let's check out our answer choices:

(A) does not get at the heart of any important assumptions in this argument. In particular, even if we negate the assumption indicated by (A), the argument stands. This isn't about overall total criticism but rather the percentage of criticism for each candidate.

(B) gets at something that might have made you uneasy when you originally read this argument - namely the author's assumption that it must be the case if one candidate received more criticism than another that such was due only to biases - assuming effectively that there was no other plausible explanation. This answer choice gets at that, so let's keep it.

(C) is out of scope - the author doesn't pronounce on whether such bias is good or bad (it might be implied, but this is the LSAT so we stick to what's written and that alone).

(D) again is out of scope- we are only concerned with reporters' biases and voting behavior - not the general electorate's voting patterns.

(E) is out of scope *and* it actually would require an assumption that would weaken the conclusion rather than strengthen it, so this is definitely not our answer.

Hence, our correct answer is (B) - any questions? :)
User avatar
 
LSAT-Chang
Thanks Received: 38
Atticus Finch
Atticus Finch
 
Posts: 479
Joined: June 03rd, 2011
 
 
trophy
Most Thankful
trophy
First Responder
 

Re: Q16 - In the last election, 89 percent

by LSAT-Chang Mon Sep 05, 2011 7:54 pm

Hi aileen, I am a little confused with (A) and (B). I completely understand what (B) means, but what I don't understand is why this would be a flaw. Even if there was more negative news worthy of reporting concerning the challenger than there was concerning the incumbent, it was the reporters who had control of news coverage, so they could have had total control in "picking" among those negative news and decide which to put MORE of. Like if there were 100 negative news of the challenger and only 50 negative news of the incumbent, the reporters have the ability to just use all 50 from the incumbent and maybe just 20 from the challenger. So I am thinking that they were biased in this sense. Does this make sense? Also, my reason as to why I think (A) is correct is because 10 pages could have been devoted to the challenger, whereas, 100 pages was devoted to the incumbent, and if this is the case, then there is technically more negative coverage of the incumbent than of the challenger, so there really is no personal bias involved. Do you see my point? Flaw questions always give me a headache because I am rarely able to "see" what the flaw is before moving on to the answer choices -- but with this case, I really thought it had to do with the amount of coverage.
 
zhanga
Thanks Received: 12
Forum Guests
 
Posts: 18
Joined: July 13th, 2011
 
This post thanked 2 times.
 
 

Re: Q16 - In the last election, 89 percent

by zhanga Wed Sep 14, 2011 12:09 pm

To answer your question:

A) is incorrect because the stimulus states that out of the coverage they were given 54% of the coverage concerning the challenger was negative and 30% concerning the incumbent was negative. The stimulus is not assuming that they received equal amounts of coverage. It doesn't matter if they weren't receiving equal coverage either because the stimulus is arguing a bias is displayed by the kind of coverage given, not the amount of coverage given.

B) is correct because it points out something the author of the stimulus is ignoring, namely the fact that there's more negative things to report on the challenger than there is on the incumbent. This might not necessarily be true, but failing to address this fact points out a weakness in the reasoning of the stimulus because it automatically attributes the unequal distribution of negative coverage to bias rather than accurate reporting.

Does this help answer your question?
 
joseph.m.kirby
Thanks Received: 55
Forum Guests
 
Posts: 70
Joined: May 07th, 2011
 
 
 

Re: Q16 - In the last election, 89 percent

by joseph.m.kirby Fri Sep 28, 2012 6:46 pm

This argument puts forward causation:

1. 89% of reporters supported the incumbent last election
2. 54% of coverage of the challenger was negative
3. 30% of coverage of the incumbent was negative
----------
C: the bias stemming from the last election CAUSED the reporters to produce more bad news about the challenger.

Overall, there's no support for bias being the cause of the bad news coverage. Everything could just be a coincidence. (B) puts forward what this argument fails to consider.
 
wgutx08
Thanks Received: 8
Elle Woods
Elle Woods
 
Posts: 52
Joined: June 09th, 2013
 
 
 

Re: PT 47 S 3 Q 16 In the last election, 89 percent

by wgutx08 Fri Sep 13, 2013 1:59 pm

aileenann Wrote:(E) is out of scope *and* it actually would require an assumption that would weaken the conclusion rather than strengthen it, so this is definitely not our answer.


Thanks so much for your kind help aileenann! However, I'm not sure which assumption you mean here and why it is "weaken rather than strengthen". I can't see why E is out of scope or the other way around. The argument says that "the reporters allowed the personal biases reflected in this voting pattern to affect their coverage..." this said bias is clearly about their personal preference, and thus different from the bias mentioned in E, a professional intention to "keep access to the incumbents".

So yeah, E looks like a very valid alternative explanation for me right now and I can't find a good reason to eliminate it. (although I actually picked B, since it is clearly correct in this context and is one of the standard LSAT flaws)

Would appreciate it so much if you (or any other LSAT geeks/peers) can help me on this one! Many thanks in advance!
 
sportsfan8491
Thanks Received: 12
Forum Guests
 
Posts: 22
Joined: August 28th, 2013
 
This post thanked 1 time.
 
 

Re: Q16 - In the last election, 89 percent

by sportsfan8491 Thu Oct 17, 2013 5:19 pm

I think I can shed some light and provide a little more clarification on what was being said in the previous post about answer choice (E), from a relevance point of view.

I don't think I can answer your strengthen or weaken question very well, because I just don't see this answer choice as being relevant. I mean, if you make some assumptions, then maybe it might have some effect on the argument. For example, one pretty significant assumption you'd need to make would be that fear of losing access would somehow affect the reporters' voting preferences and, thus, skew their news coverage reporting somewhat in favor of the incumbent. But without making this pretty big leap, I just don't see it having any impact from a strengthen or weaken perspective, especially given that this argument seems to be focusing in on the personal "voting preferences" of reporters as being the main reason behind the bias in the news reporting.

So, I ruled out this answer for the first reason mentioned by aileenann, which is that of relevance. Nowhere in the stimulus is the comparison made that the reporters fear something happening "MORE SO" than something else.

More importantly, the combination/synthesis of the first two sentences makes it seem as though personal bias refers to the voting preferences of the reporters and I don't see how fear of losing access would influence the reporters' voting preferences. Fear of losing access, in my opinion, wouldn't have anything to do with why the reporters voted in a particular way. I think the tricky thing about this question is that it might be easy to misconstrue "personal biases" if one doesn't pay close attention to the qualifying phrase right after this, which states "reflected in THIS voting pattern". Thus, it's not talking about just any biases, but it puts a constraint on the type of bias it's actually discussing. "THIS voting pattern" appears to be referential phrasing that points to the "89 percent...voted FOR the incumbent" in the first sentence, which is why I believe this stimulus is talking about the reporters' voting preferences and why fear of losing access is completely irrelevant to this discussion.

I think the "ignores the possibility" portion makes this answer even trickier to rule out, but just because it ignores this possibility doesn't mean that it is a flaw that this argument commits. The argument ignores a lot of other possibilities as well.

I hope this helps.
 
christine.defenbaugh
Thanks Received: 585
Atticus Finch
Atticus Finch
 
Posts: 536
Joined: May 17th, 2013
 
 
 

Re: Q16 - In the last election, 89 percent

by christine.defenbaugh Wed Oct 23, 2013 1:48 am

Great question wgutx08, and lots of good thoughts here sportsfan8491!

wgutx08 brings up a really excellent point, and one that can't be dismissed out of hand.

Let's return to the core briefly:

Premises:
Reporters voted 89% for I.
Of the coverage they received, a higher percent was negative for C than for I.

Conclusion:
The voting biases of the reporters caused that difference.

Necessary Assumption:
Nothing else caused that difference.

Both (B) and (E) start with "ignores the possibility that", and so should contain an ignored possibility that breaks that necessary assumption. Ideally, we want an alternative cause for the negative-reporting-difference, so that we can take the heat off the reporter's personal biases.

(B) clearly gives us one. If there were more negative news about the challenger that was objectively worthy of reporting, that could easily explain the negative-reporting-difference.

But (E) feels like it offers another alternative cause. After all, if reporters are afraid of losing access to incumbents, and if more negative reporting about an incumbent ran a risk of reducing the access to that incumbent, perhaps reporters would be adjusting the news for *that* reason, instead of as a result of their personal political inclinations.

But pay careful attention to the thought chain there. If reporters are afraid of losing access to incumbents and if more negative reporting would risk that access... We added an additional assumption to the possibility in (E) in order to make it break the assumption. We can't do that! We have to add in some connective glue to (E) to get it to connect to the negative-reporting-difference, and that means that on its own it does not break the assumption.

[sportsfan8491, note that (E) does not have to cause or explain the reporters' voting patterns. The conclusion is that political bias caused the negative-reporting-difference, so all (E) would need to do is offer an alternative explanation of that.]

Some great discussion here! Please let me know if you have additional questions on this!
 
LukeM22
Thanks Received: 6
Elle Woods
Elle Woods
 
Posts: 53
Joined: July 23rd, 2017
 
 
 

Re: Q16 - In the last election, 89 percent

by LukeM22 Mon Nov 13, 2017 11:44 pm

I'm probably overthinking this, but I've gone through the above analysis and I'm still at a 50-50 split between B and E.

First, I agree that E does require an additional premise/leap-- that there is some connection between access to the candidates and positive/negative coverage of those candidates. That being said, I wrote off B because it seemed "more negative news worthy of reporting concerning the challenger than there was concerning the incumbent" could have two interpretations:

1) The absolute value/overall quantity of content of negative news for the challenger was greater than that of the incumbent-- that is, there were simply more negative stories for guy A than guy B. However, we still don't know the proportion of negative to positive news for each of the candidates, which is what the stimulus was concerned with, not the overall number of stories/content. The challenger having 200 negative stories vs. the incumbent's 100 (despite the challenger having 1000 positive stories vs. the incumbent's 200 positive stories) would work within this universe but not really provide an adequate explanation.

2) The proportion of negative to positive stories was greater for challenger than incumbent. This would adequately explain the phenomenon, and is what everyone else's interpretation of this appears to be, and is obviously the right answer, but it would only follow if we knew anything about the amount of positive news for either candidate, which we don't.

I don't know, I just felt something about the way "more" was presented in B's answer choice was ambiguous enough to the point where it could have multiple interpretations (some of which wouldn't really work), and for that reason, E was no worse of an answer choice than B. What is off with my analysis? I've been noticing this problem multiple times.

Edit: I had this same problem is Question 8, where "most often" referred to an absolute quantity, and "more", here, refers to a proportion. Is there a hard and fast rule re: when I should know what refers to what?

Thanks for the help!
User avatar
 
ohthatpatrick
Thanks Received: 3808
Atticus Finch
Atticus Finch
 
Posts: 4661
Joined: April 01st, 2011
 
 
 

Re: Q16 - In the last election, 89 percent

by ohthatpatrick Tue Nov 14, 2017 2:46 pm

Great thoughts!

I would interpret both of those (Q8 and Q16) as quantity, not proportion.

The fact that there is a higher quantity of negative news for the challenger does not PROVE that the proportion of neg/pos was worse for the challenger, but it supports that idea.

(B) and (E) are both just offering us potential objections in the form of "you're ignoring this ALTERNATE EXPLANATION for the 54% vs. 30% asymmetry"

So just ask yourself which one LSAT considers a stronger (not perfect) alternate explanation:
The challenger got more negative coverage because reporters generally fear losing access to incumbents more than to challengers
or
The challenger got more negative coverage than the incumbent because THIS challenger had more negative news worthy of reporting than THIS incumbent.

Since neither alternate explanation is bulletproof, pick the one that's more germane, more specific to our situation (this incumbent, this challenger vs. most incumbents/challengers ) and more specific to what we're trying to explain.

"How much negative news was worthy of reporting"
is more explicitly connected to
"54% vs. 30% negative news coverage"
than is
"how fearful a reporter is of losing access to a candidate"